1
2
3
4
5
6
7
8
9
10
11
12
13
14
15
16
17
18
19
20
21
22
23
24
25
26
27
28
29
30
31
32
33
34
35
36
37
38
39
40
41
42
43
44
45
46
47
48
49
50
51
52
53
54
55
56
57
58
59
60
61
62
63
64
65
66
67
68
69
70
71
72
73
74
75
76
77
78
79
80
81
82
83
84
85
86
87
88
89
90
91
92
93
94
95
96
97
98
99
100
101
102
103
104
105
106
107
108
109
110
111
112
113
114
115
116
117
118
119
120
121
122
123
124
125
126
127
128
129
130
131
132
133
134
135
136
137
138
139
140
141
142
143
144
145
146
147
148
149
150
151
152
153
154
155
156
157
158
159
160
161
162
163
164
165
166
167
168
169
170
171
172
173
174
175
176
177
178
179
180
181
182
183
184
185
186
187
188
189
190
191
192
193
194
195
196
197
198
199
200
201
202
203
204
205
206
207
208
209
210
211
212
213
214
215
216
217
218
219
220
221
222
223
224
225
226
227
228
229
230
231
232
233
234
235
236
237
238
239
240
241
242
243
244
245
246
247
248
249
250
251
252
253
254
255
256
257
258
259
260
261
262
263
264
265
266
267
268
269
270
271
272
273
274
275
276
277
278
279
280
281
282
283
284
285
286
287
288
289
290
291
292
293
294
295
296
297
298
299
300
301
302
303
304
305
306
307
308
309
310
311
312
313
314
315
316
317
318
319
320
321
322
323
324
325
326
327
328
329
330
331
332
333
334
335
336
337
338
339
340
341
342
343
344
345
346
347
348
349
350
351
352
353
354
355
356
357
358
359
360
361
362
363
364
365
366
367
368
369
370
371
372
373
374
375
376
377
378
379
380
381
382
383
384
385
386
387
388
389
390
391
392
393
394
395
396
397
398
399
400
401
402
403
404
405
406
407
408
409
410
411
412
413
414
415
416
417
418
419
420
421
422
423
424
425
426
427
428
429
430
431
432
433
434
435
436
437
438
439
440
441
442
443
444
445
446
447
448
449
450
451
452
453
454
455
456
457
458
459
460
461
462
463
464
465
466
467
468
469
470
471
472
473
474
475
476
477
478
479
480
481
482
483
484
485
486
487
488
489
490
491
492
493
494
495
496
497
498
499
500
501
502
503
504
505
506
507
508
509
510
511
512
513
514
515
516
517
518
519
520
521
522
523
524
525
526
527
528
529
530
531
532
533
534
535
536
537
538
539
540
541
542
543
544
545
546
547
548
549
550
551
552
553
554
555
556
557
558
559
560
561
562
563
564
565
566
567
568
569
570
571
572
573
574
575
576
577
578
579
580
581
582
583
584
585
586
587
588
589
590
591
592
593
594
595
596
597
598
599
600
601
602
603
604
605
606
607
608
609
610
611
612
613
614
615
616
617
618
619
620
621
622
623
624
625
626
627
628
629
630
631
632
633
634
635
636
637
638
639
640
641
642
643
644
645
646
647
648
649
650
651
652
653
654
655
656
657
658
659
660
661
662
663
664
665
666
667
668
669
670
671
672
673
674
675
676
677
678
679
680
681
682
683
684
685
686
687
688
689
690
691
692
693
694
695
696
697
698
699
700
701
702
703
704
705
706
707
708
709
710
711
712
713
714
715
716
717
718
719
720
721
722
723
724
725
726
727
728
729
730
731
732
733
734
735
736
737
738
739
740
741
742
743
744
745
746
747
748
749
750
751
752
753
754
755
756
757
758
759
760
761
762
763
764
765
766
767
768
769
770
771
772
773
774
775
776
777
778
779
780
781
782
783
784
785
786
787
788
789
790
791
792
793
794
795
796
797
798
799
800
801
802
803
804
805
806
807
808
809
810
811
812
813
814
815
816
817
818
819
820
821
822
823
824
825
826
827
828
829
830
831
832
833
834
835
836
837
838
839
840
841
842
843
844
845
846
847
848
849
850
851
852
853
854
855
856
857
858
859
860
861
862
863
864
865
866
867
868
869
870
871
872
873
874
875
876
877
878
879
880
881
882
883
884
885
886
887
888
889
890
891
892
893
894
895
896
897
898
899
900
901
902
903
904
905
906
907
908
909
910
911
912
913
914
915
916
917
918
919
920
921
922
923
924
925
926
927
928
929
930
931
932
933
934
935
936
937
938
939
940
941
942
943
944
945
946
947
948
949
950
951
952
953
954
955
956
957
958
959
960
961
962
963
964
965
966
967
968
969
970
971
972
973
974
975
976
977
978
979
980
981
982
983
984
985
986
987
988
989
990
991
992
993
994
995
996
997
998
999
1000
1001
1002
1003
1004
1005
1006
1007
1008
1009
1010
1011
1012
1013
1014
1015
1016
1017
1018
1019
1020
1021
1022
1023
1024
1025
1026
1027
1028
1029
1030
1031
1032
1033
1034
1035
1036
1037
1038
1039
1040
1041
1042
1043
1044
1045
1046
1047
1048
1049
1050
1051
1052
1053
1054
1055
1056
1057
1058
1059
1060
1061
1062
1063
1064
1065
1066
1067
1068
1069
1070
1071
1072
1073
1074
1075
1076
1077
1078
1079
1080
1081
1082
1083
1084
1085
1086
1087
1088
1089
1090
1091
1092
1093
1094
1095
1096
1097
1098
1099
1100
1101
1102
1103
1104
1105
1106
1107
1108
1109
1110
1111
1112
1113
1114
1115
1116
1117
1118
1119
1120
1121
1122
1123
1124
1125
1126
1127
1128
1129
1130
1131
1132
1133
1134
1135
1136
1137
1138
1139
1140
1141
1142
1143
1144
1145
1146
1147
1148
1149
1150
1151
1152
1153
1154
1155
1156
1157
1158
1159
1160
1161
1162
1163
1164
1165
1166
1167
1168
1169
1170
1171
1172
1173
1174
1175
1176
1177
1178
1179
1180
1181
1182
1183
1184
1185
1186
1187
1188
1189
1190
1191
1192
1193
1194
1195
1196
1197
1198
1199
1200
1201
1202
1203
1204
1205
1206
1207
1208
1209
1210
1211
1212
1213
1214
1215
1216
1217
1218
1219
1220
1221
1222
1223
1224
1225
1226
1227
1228
1229
1230
1231
1232
1233
1234
1235
|
{
"metadata": {
"name": "",
"signature": "sha256:b2d866884e32978f30135a4e666314d83655be533eae1227e5f3d0fa8ae8f5d8"
},
"nbformat": 3,
"nbformat_minor": 0,
"worksheets": [
{
"cells": [
{
"cell_type": "heading",
"level": 1,
"metadata": {},
"source": [
"Chapter 11: Absorption And Ion Exchange"
]
},
{
"cell_type": "heading",
"level": 2,
"metadata": {},
"source": [
"Ex11.1: Page 575"
]
},
{
"cell_type": "code",
"collapsed": false,
"input": [
"\n",
"\n",
"# Illustration 11.1\n",
"# Page: 575\n",
"\n",
"print'Illustration 11.1 - Page: 575\\n\\n'\n",
"\n",
"# Solution\n",
"import numpy\n",
"import matplotlib.pyplot as plt\n",
"%matplotlib inline\n",
"#*****Data*****#\n",
"Temp = 30.0;# [OC]\n",
"#*************#\n",
"\n",
"# From Fig. 11.5 (Pg 572)\n",
"# The isosteres for various concentrations are straight and their slopes are measured with the help of milimeter rule.\n",
"# Data = [X(kg acetone/kg carbon) lambda(slope of isostere)]\n",
"Data = numpy.array([[0.05 ,1.170],[0.10, 1.245],[0.15 ,1.3],[0.20 ,1.310],[0.25 ,1.340],[0.30 ,1.327]]);# [kg acetone/kg carbon]\n",
"lambdar = 551.0;# [reference at 30 OC,kJ/kg]\n",
"Val = numpy.zeros(shape=(6,5));\n",
"for i in range(0,6):\n",
" Val[i,0] = Data[i,0];# [kg acetone/kg carbon]\n",
" Val[i,1] = Data[i,1];# [slope of isostere]\n",
" Val[i,2] = -Data[i,1]*lambdar;# [kJ/kg acetone]\n",
"\n",
"\n",
"plt.plot(Val[:,0],Val[:,2])\n",
"plt.grid();\n",
"xlabel(\"X (kg carbon / kg acetone)\");\n",
"ylabel(\"Differential heat of adsorption (kJ / kg acetone)\");\n",
"title(\"Graphical Integration\");\n",
"plt.show()\n",
"# Area: The area under the curve between X = 0 to X = X\n",
"# Corresponding to Data(:,1):\n",
"Area = numpy.array([-29.8 ,-63.0, -97.9 ,-134.0, -170.5, -207.5]);\n",
"for i in range(0,6):\n",
" Val[i,3] = Area[i];\n",
" Val[i,4] = Area[i]+(lambdar*Val[i,0]);\n",
"print \" (1) = X(kg acetone/kg carbon) \\n (2)= Slope of isostere \\n (3)= Differential heat of adsorption(kJ/kg acetone) \\n (4)=deltaH_prime(vapour(kJ/kg carbon)) \\n (5)=deltaH(liquid(kJ/kg carbon)\"\n",
"print\"(1) \\t \\t \\t \\t (2) \\t \\t \\t \\t (3) \\t \\t \\t \\t \\t \\t \\t \\t (4) \\t \\t \\t \\t \\t \\t (5) \" \n",
"for i in range(0,6):\n",
" print Val[i,0],\" \\t \\t \\t \",Val[i,1],\" \\t \\t \",Val[i,2],\" \\t \\t \\t \\t \\t \",Val[i,3],\" \\t \\t \\t \\t\",Val[i,4]\n",
"#the answers are slightly different in textbook due to approximation while here answers are precise"
],
"language": "python",
"metadata": {},
"outputs": [
{
"output_type": "stream",
"stream": "stdout",
"text": [
"Illustration 11.1 - Page: 575\n",
"\n",
"\n"
]
},
{
"metadata": {},
"output_type": "display_data",
"png": "iVBORw0KGgoAAAANSUhEUgAAAZQAAAEZCAYAAACw69OmAAAABHNCSVQICAgIfAhkiAAAAAlwSFlz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\n",
"text": [
"<matplotlib.figure.Figure at 0x76f12b0>"
]
},
{
"output_type": "stream",
"stream": "stdout",
"text": [
" (1) = X(kg acetone/kg carbon) \n",
" (2)= Slope of isostere \n",
" (3)= Differential heat of adsorption(kJ/kg acetone) \n",
" (4)=deltaH_prime(vapour(kJ/kg carbon)) \n",
" (5)=deltaH(liquid(kJ/kg carbon)\n",
"(1) \t \t \t \t (2) \t \t \t \t (3) \t \t \t \t \t \t \t \t (4) \t \t \t \t \t \t (5) \n",
"0.05 \t \t \t 1.17 \t \t -644.67 \t \t \t \t \t -29.8 \t \t \t \t-2.25\n",
"0.1 \t \t \t 1.245 \t \t -685.995 \t \t \t \t \t -63.0 \t \t \t \t-7.9\n",
"0.15 \t \t \t 1.3 \t \t -716.3 \t \t \t \t \t -97.9 \t \t \t \t-15.25\n",
"0.2 \t \t \t 1.31 \t \t -721.81 \t \t \t \t \t -134.0 \t \t \t \t-23.8\n",
"0.25 \t \t \t 1.34 \t \t -738.34 \t \t \t \t \t -170.5 \t \t \t \t-32.75\n",
"0.3 \t \t \t 1.327 \t \t -731.177 \t \t \t \t \t -207.5 \t \t \t \t-42.2\n"
]
}
],
"prompt_number": 1
},
{
"cell_type": "heading",
"level": 2,
"metadata": {},
"source": [
"Ex11.2: Page 596"
]
},
{
"cell_type": "code",
"collapsed": false,
"input": [
"\n",
"\n",
"# Illustration 11.2\n",
"# Page: 596\n",
"\n",
"print'Illustration 11.2 - Page: 596\\n\\n'\n",
"\n",
"# solution\n",
"import numpy\n",
"import matplotlib.pyplot as plt\n",
"%matplotlib inline\n",
"#*****Data*****#\n",
"# x:kg carbon/kg soln\n",
"# y_star: Equilibrium colour, units/kg soln.\n",
"# X:adsorbate concentration, units/kg carbon\n",
"# Data = [x Y_star]\n",
"Data =numpy.array([[0, 9.6],[0.001, 8.6],[0.004 ,6.3],[0.008, 4.3],[0.02 ,1.7],[0.04, 0.7]]);\n",
"Yo = 9.6;# [units of colour/kg soln]\n",
"Y1 = 0.1*Yo;# [units of colour/kg soln]\n",
"Ls = 1000.0;# [kg soln]\n",
"#****************#\n",
"\n",
"\n",
"n = 1.66;# [slope of line]\n",
"# At X = 663, Y_star = 4.3\n",
"# From eqn. 11.5\n",
"X = 663;\n",
"Y_star = 4.3;\n",
"m = Y_star/X**n;\n",
"# Freundlich Equation:\n",
"def f76(X):\n",
" return m*X**n\n",
"X = numpy.arange(0,1000,1);\n",
"\n",
"plt.plot(X,f76(X));\n",
"plt.grid('on');\n",
"plt.xlabel(\"units of colour/kg carbon\");\n",
"plt.ylabel(\"units of colour/kg solution\");\n",
"title(\"Equilibium Data(on arithmetic scale)\");\n",
"plt.show()\n",
"# Single Stage Operation:\n",
"# Since fresh carbn is used:\n",
"Xo = 0;# [units/kg carbon]\n",
"# From scf(30):\n",
"X1 = 270;# [units/kg carbon]\n",
"Data2 =numpy.array([[Xo, Yo],[X1, Y1]]);\n",
"\n",
"plt.plot(X,f76(X),label=\"Equilbrium curve\")\n",
"plt.plot(Data2[:,0],Data2[:,1],label=\"Operating line curve\")\n",
"plt.grid('on');\n",
"plt.xlabel(\"units of colour/kg carbon\");\n",
"plt.ylabel(\"units of colour/kg solution\");\n",
"plt.legend(loc='upper left');\n",
"plt.title(\"Single stage operation\");\n",
"plt.show()\n",
"# From Eqn. 11.4:\n",
"Ss = Ls*((Yo-Y1)/(X1-Xo));# [kg carbon/kg soln]\n",
"print\"Quantity of fresh carbon recquired for single stage operation: \",Ss,\" kg carbon/1000 kg solution\\n\"\n",
"\n",
"# Two stage cross current operation:\n",
"# For the minimumamount of carbon:\n",
"X1 = 565;# [units/kg carbon]\n",
"Y1 = 3.30;# [units of colour/kg soln]\n",
"X2 = 270;# [units/kg carbon]\n",
"Y2 = 0.96;# [units of colour/kg soln]\n",
"Data3 = numpy.array([[Xo ,Yo],[X1 ,Y1]]);\n",
"Data4 = numpy.array([[0 ,Y1],[X2 ,Y2]]);\n",
"\n",
"plt.plot(X,f76(X),label=\"Equilbrium curve\")\n",
"plt.plot(Data3[:,0],Data3[:,1],label=\"First of two Cocurrent\")\n",
"plt.plot(Data4[:,0],Data4[:,1],label=\"Second of two Cocurrent\")\n",
"plt.grid('on');\n",
"plt.xlabel(\"units of colour/kg carbon\");\n",
"plt.ylabel(\"units of colour/kg solution\");\n",
"plt.legend(loc='upper left');\n",
"plt.title(\"Two stage Cross current operation\");\n",
"plt.show()\n",
"# From Eqn. 11.8:\n",
"Ss1 = Ls*(Yo-Y1)/(X1-Xo);# [kg]\n",
"Ss2 = Ls*(Y1-Y2)/(X2-Xo);# [kg]\n",
"Ss = Ss1+Ss2;# [kg]\n",
"print\"Quantity of fresh carbon recquired for two stage crosscurrent operation: \",Ss,\" kg carbon/1000 kg solution\\n\"\n",
"\n",
"# Two Stage counter current operation:\n",
"Yo = 9.6;\n",
"Y2 = 0.96;\n",
"# By trial and error:\n",
"XNpPlus1 = 0;\n",
"X1 = 675;\n",
"Data5 = numpy.array([[X1 ,Yo],[XNpPlus1 ,Y2]]);\n",
"\n",
"plt.plot(X,f76(X),label=\"Equilbrium curve\")\n",
"plt.plot(Data5[:,0],Data5[:,1],label=\"Two stage Counter Current\");\n",
"plt.grid('on');\n",
"plt.xlabel(\"units of colour/kg carbon\");\n",
"plt.ylabel(\"units of colour/kg solution\");\n",
"plt.legend(loc='upper left');\n",
"plt.title(\"Two stage Counter Current operation\");\n",
"# By eqn 11.14:\n",
"Ss = Ls*(Yo-Y2)/(X1-XNpPlus1);\n",
"print\"Quantity of fresh carbon recquired for two stage Counter Current operation: \",Ss,\" kg carbon/1000 kg solution\\n\""
],
"language": "python",
"metadata": {},
"outputs": [
{
"output_type": "stream",
"stream": "stdout",
"text": [
"Illustration 11.2 - Page: 596\n",
"\n",
"\n"
]
},
{
"metadata": {},
"output_type": "display_data",
"png": "iVBORw0KGgoAAAANSUhEUgAAAYEAAAEZCAYAAABxbJkKAAAABHNCSVQICAgIfAhkiAAAAAlwSFlz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\nAAAAAElFTkSuQmCC\n",
"text": [
"<matplotlib.figure.Figure at 0x76f16d8>"
]
},
{
"metadata": {},
"output_type": "display_data",
"png": "iVBORw0KGgoAAAANSUhEUgAAAYYAAAEZCAYAAACTsIJzAAAABHNCSVQICAgIfAhkiAAAAAlwSFlz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\nTA6WGIwxxuRgicEYY0wOlhiMMcbk8P+dWPmzYQm/hQAAAABJRU5ErkJggg==\n",
"text": [
"<matplotlib.figure.Figure at 0x7930c18>"
]
},
{
"output_type": "stream",
"stream": "stdout",
"text": [
"Quantity of fresh carbon recquired for single stage operation: 32.0 kg carbon/1000 kg solution\n",
"\n"
]
},
{
"metadata": {},
"output_type": "display_data",
"png": "iVBORw0KGgoAAAANSUhEUgAAAYYAAAEZCAYAAACTsIJzAAAABHNCSVQICAgIfAhkiAAAAAlwSFlz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\nEwAAAABJRU5ErkJggg==\n",
"text": [
"<matplotlib.figure.Figure at 0x7938c88>"
]
},
{
"output_type": "stream",
"stream": "stdout",
"text": [
"Quantity of fresh carbon recquired for two stage crosscurrent operation: 19.8171091445 kg carbon/1000 kg solution\n",
"\n",
"Quantity of fresh carbon recquired for two stage Counter Current operation: "
]
},
{
"output_type": "stream",
"stream": "stdout",
"text": [
" 12.8 kg carbon/1000 kg solution\n",
"\n"
]
},
{
"metadata": {},
"output_type": "display_data",
"png": "iVBORw0KGgoAAAANSUhEUgAAAYYAAAEZCAYAAACTsIJzAAAABHNCSVQICAgIfAhkiAAAAAlwSFlz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\n",
"text": [
"<matplotlib.figure.Figure at 0x7d9ad68>"
]
}
],
"prompt_number": 2
},
{
"cell_type": "heading",
"level": 2,
"metadata": {},
"source": [
"Ex11.3: Page 602"
]
},
{
"cell_type": "code",
"collapsed": false,
"input": [
"\n",
"\n",
"# Illustration 11.3\n",
"# Page: 602\n",
"\n",
"print'Illustration 11.3 - Page: 602\\n\\n'\n",
"\n",
"# Solution\n",
"import numpy\n",
"from scipy.optimize import fsolve\n",
"import math\n",
"#***Data***#\n",
"T = 1.0; #[m]\n",
"di = 0.203;# [m]\n",
"n = 1;# [for one impeller]\n",
"Density_S = 2300.0;# [kg/cubic m]\n",
"Density_p = 2300.0;# [kg/cubic m]\n",
"C = 0.150;# [m]\n",
"S = 50.0;# [kg]\n",
"g = 9.807;# [m/s]\n",
"dp = 8*10**(-4);# [m]\n",
"N = 8.33; #[r/s]\n",
"Temp=25;# [OC]\n",
"#*************#\n",
"\n",
"# Assume:\n",
"Po = 5;\n",
"viscosity_L = 8.94*10**(-4);# [kg/m.s]\n",
"Density_L = 998.0;# [kg/cubic m]\n",
"delta_Density = Density_S-Density_L;# [kg/cubic m]\n",
"# By Eqn. 11.23:\n",
"Vts = g*dp**2*delta_Density/(18*viscosity_L);# [m/s]\n",
"# By defn. of power number:\n",
"# P = Po*Density_m*di**5*Ni**3\n",
"# vm = math.pi*T**2*(Z+C)/4\n",
"# Solid Volume = S/Density_p;\n",
"# If these are substituted in Eqn. 11.22\n",
"def f(Z):\n",
" return (((Z+C)**(1.3/3))*math.exp(4.35*Z/(T-0.1)))-((1.0839*Po*di**(11.0/2)*N**3*Density_p**(2.0/3))/(g*Vts*T**(7.0/6)*S**(2.0/3)))\n",
"Z = fsolve(f,7);# [m]\n",
"phi_Sm = 4*S/(math.pi*T**2*(Z+C)*Density_p);\n",
"Density_m = (phi_Sm*Density_p)+((1-phi_Sm)*Density_L);# [kg/cubic m]\n",
"phi_Ss = 0.6;\n",
"viscosity_m = viscosity_L/(1-(phi_Sm/phi_Ss))**1.8;# [kg/m.s]\n",
"Re = di**2*N*Density_m/viscosity_m;\n",
"P = Po*Density_m*N**3*di**5;# [W]\n",
"print \"Agitator Power required: \",round(P),\" W\\n\"\n",
"#the answers are slightly different in textbook due to approximation while here answers are precise"
],
"language": "python",
"metadata": {},
"outputs": [
{
"output_type": "stream",
"stream": "stdout",
"text": [
"Illustration 11.3 - Page: 602\n",
"\n",
"\n",
"Agitator Power required: 1113.0 W\n",
"\n"
]
}
],
"prompt_number": 65
},
{
"cell_type": "heading",
"level": 2,
"metadata": {},
"source": [
"Ex11.4: Page 604"
]
},
{
"cell_type": "code",
"collapsed": false,
"input": [
"\n",
"\n",
"# Illustration 11.4\n",
"# Page: 604\n",
"\n",
"print'Illustration 11.4 - Page: 604\\n\\n'\n",
"\n",
"import math\n",
"#****Data*****#\n",
"# b: kerosene c:water\n",
"# c:kg water/cubic m liquid\n",
"Density_l = 783;# [kg/cubic m]\n",
"viscosity_l = 1.7*10**(-3);# [kg/m.s]\n",
"Mb = 200;# [kg/kmol]\n",
"Density_p = 881;# [kg/cubic m]\n",
"m = 0.522;# [(kg water/cubic m kerosene)/(kg water/kg gel)]\n",
"Xo = 0;# [kg H2O/kg gel]\n",
"#**************#\n",
"\n",
"# Solution (a)\n",
"co = Density_l*4*10**(-5);# [kg water/cubic m]\n",
"c1 = Density_l*5*10**(-6);# [kg water/cubic m]\n",
"# For Ss minimum:\n",
"X1 = c1/m;# [kg H2O/kg gel]\n",
"# By Water Balance:\n",
"SsminByVl = (co-c1)/(X1-Xo);# [kg gel/cubic m kerosene]\n",
"print\"Minimum Solid/Liquid ratio used:\",SsminByVl,\" kg gel/cubic m kerosene\"\n",
"print\"\\n\"\n",
"\n",
"# Solution (b)\n",
"# Basis: 1 batch,1.7 cubic m kerosene\n",
"Vl = 1.7;# [cubic m]\n",
"Ss = 16*1.7;# [kg gel]\n",
"V = Ss/Density_p;# [Xol. solid, cubic m]\n",
"Vt = 1.7+V;# [Total batch volume, cubic m]\n",
"# Take Z = T\n",
"T = (Vt*4/math.pi)**(1.0/3);# [m]\n",
"# To allow for the adequate free board:\n",
"h = 1.75;# [Vessel height,m]\n",
"# Use a six-blade disk impeller.\n",
"# From Fig. 11.26:\n",
"# dp corresponding to 14 mesh:\n",
"dp = 1.4/1000;# [m]\n",
"TBydi1 = 2.0;\n",
"Value1 = (Density_p-Density_l)/Density_l;\n",
"# From Fig. 11.26:\n",
"TBydi2 = 4.4;\n",
"TBydiAv = (TBydi1+TBydi2)/2.0;\n",
"di = T/TBydiAv;# [m]\n",
"fr = 0.6;# [settled volume fraction of solids]\n",
"Vs = V/fr;# [cubic m]\n",
"depth = Vs/((math.pi*(T**2))/4);# [m]\n",
"# The depth of settled solid is negligible.\n",
"# Locate the turbine 150mm from the bottom of the tank.\n",
"C = 0.150;# [m]\n",
"\n",
"# Power:\n",
"# Use the sufficient agitator power to lift the solids to 0.6 m above the bottom of the vessel.\n",
"Z_prime = 0.6-C;# [m]\n",
"# The properties of the slurry in 0.6 m above the bottom of the vessel.\n",
"Vm = 0.6*math.pi*T**2.0/4;# [square m]\n",
"phi_Sm = V/Vm;# [vol fraction solid]\n",
"# From Eqn. 11.24:\n",
"Density_m = (phi_Sm*Density_p)+((1-phi_Sm)*Density_l);# [kg/cubic m]\n",
"# From Eqn. 11.25:\n",
"phi_Ss = 0.8;\n",
"viscosity_m = viscosity_l/(1-(phi_Sm/phi_Ss))**1.8;# [kg/m.s]\n",
"g = 9.81;# [m/s^2]\n",
"# From Eqn. 11.23:\n",
"delta_Density = Density_p-Density_l;# [kg/cubic m]\n",
"Vts = g*dp**2*delta_Density/(18*viscosity_l);# [m/s]\n",
"# From Eqn. 11.22:\n",
"n = 1.0;\n",
"P = (g*n*Density_m*Vm*Vts)*(phi_Sm**(2.0/3))*(TBydiAv**(1.0/2))*math.exp((4.35*Z_prime/T)-0.1);# [W]\n",
"# Assume:\n",
"Po = 5.0;\n",
"N = (P/(Po*Density_m*di**5))**(1.0/3);# [r/s]\n",
"# Use:\n",
"N1 = 2.0;# [r/s]\n",
"Re = di**2.0*N1*Density_m/viscosity_m;\n",
"# From fig. 6.5: Po = 5\n",
"# Hence our assumption was right.\n",
"print\"Power delivered to the slurry: \",round((P*(N1/N)**3),2),\" W\\n\",\n",
"print\"Power to the motor will be larger, depending on the efficiency of the motor and speed reducer.\\n\"\n",
"\n",
"# Mass transfer: \n",
"# From Eqn. 11.28:\n",
"Rep = (dp**(4.0/3))*(P/Vl)**(1.0/3)*(Density_l**(2.0/3)/viscosity_l);\n",
"# From Eqn. 2.44:\n",
"Temp = 298;# [K]\n",
"phi = 1.0;\n",
"Va = 0.0756;# [Chapter 2 notation]\n",
"Dl = ((117.3*10**(-18))*((phi*Mb)**0.5)*Temp)/(viscosity_l*(Va**(0.6)));\n",
"ScL = viscosity_l/(Density_l*Dl);\n",
"if dp<(2.0/1000):\n",
" # From Eqn. 11.29:\n",
" ShL = 2+(0.47*Rep**0.62*(1/TBydiAv**0.17)*ScL**0.36);\n",
"else:\n",
" # From Eqn. 11.30:\n",
" ShL = 0.222*Rep**(3.0/4)*ScL**(1.0/3);\n",
"\n",
"kL = ShL*Dl/dp;# [m/s]\n",
"apS = (math.pi*dp**2)/(math.pi*dp**3*Density_p/6.0);\n",
"apL = apS*16;# [square m/cubic m liquid]\n",
"Ratio = Ss/(Vl*m);\n",
"# From Eqn. 11.40:\n",
"thetha = math.log((co/c1)/(1+(1/Ratio)-(1/Ratio)*(co/c1)))/((1+(1/Ratio))*kL*apL);\n",
"print\"Contacting Time required: \",round(thetha/60,2),\" min\\n\"\n",
"#the answers are slightly different in textbook due to approximation while here answers are precise"
],
"language": "python",
"metadata": {},
"outputs": [
{
"output_type": "stream",
"stream": "stdout",
"text": [
"Illustration 11.4 - Page: 604\n",
"\n",
"\n",
"Minimum Solid/Liquid ratio used: 3.654 kg gel/cubic m kerosene\n",
"\n",
"\n",
"Power delivered to the slurry: 350.05 W\n",
"Power to the motor will be larger, depending on the efficiency of the motor and speed reducer.\n",
"\n",
"Contacting Time required: 8.3 min\n",
"\n"
]
}
],
"prompt_number": 69
},
{
"cell_type": "heading",
"level": 2,
"metadata": {},
"source": [
"Ex11.5: Page 606"
]
},
{
"cell_type": "code",
"collapsed": false,
"input": [
"\n",
"\n",
"# Illustration 11.5\n",
"# Page: 606\n",
"\n",
"print'Illustration 11.5 - Page: 606\\n\\n'\n",
"\n",
"# Solution\n",
"\n",
"import math\n",
"import numpy.linalg as lin\n",
"#*****Data******#\n",
"Vl = 1.1*10**(-4);# [cubic m/s]\n",
"Ss = 0.0012;# [kg/s]\n",
"Density_p = 1120;# [kg/cubic m]\n",
"dp = 8*10**(-4);# [m]\n",
"Ds = 2*10**(-11);# [square m/s]\n",
"Dl = 7.3*10**(-10);# [square m/s]\n",
"m = 0.2;# [(kg Cu2+/cubic m soln)/(kg Cu2+/kg resin)]\n",
"T = 1;# [m]\n",
"#********************#\n",
"\n",
"Z = T;# [m]\n",
"# The particles will be lifted to the top of the vessel.\n",
"Z_prime = 0.5;# [m]\n",
"viscosity_l = 8.94*10**(-4);# [kg/m.s]\n",
"Density_l = 998;# [kg/cubic m]\n",
"delta_Density = Density_p-Density_l;# [kg/cubic m]\n",
"g = 9.80;# [m/square s]\n",
"# From Eqn. 11.23:\n",
"Vts = g*dp**2*delta_Density/(18*viscosity_l);\n",
"Vm = math.pi*T**2*Z/4.0;# [cubic m]\n",
"Vs = Ss/Density_p;# [cubic m/s]\n",
"phi_Sm = Vs/(Vs+Vl);# [vol fraction]\n",
"# From eqn. 11.24:\n",
"Density_m = (phi_Sm*Density_p)+((1-phi_Sm)*Density_l);# [kg/cubic m]\n",
"# From Eqn. 11.22:\n",
"n = 1.0;\n",
"di = 0.3;# [m]\n",
"P = (g*n*Density_m*Vm*Vts)*(phi_Sm**(2.0/3))*((T/di)**(1.0/2))*math.exp((4.35*Z_prime/T)-0.1);# [W]\n",
"# To estimate the impeller speed:\n",
"# Assume:\n",
"Po = 5;\n",
"N = (P/(Po*Density_m*di**5))**(1.0/3);# [r/s]\n",
"Re = di**2*N*Density_m/viscosity_l;\n",
"# From fig. 6.5: Assumption of Po was correct.\n",
"print\"Speed of the impeller:\",round(N,2),\" r/s\\n\"\n",
"vT = (math.pi/4.0)*T**2*Z;# [cubic m]\n",
"vL = vT*(1-phi_Sm);\n",
"# From Eqn. 11.28:\n",
"Rep = (dp**(4.0/3))*(P/vL)**(1.0/3)*(Density_l**(2.0/3)/viscosity_l);\n",
"ScL = viscosity_l/(Density_l*Dl);\n",
"if dp<(2.0/1000):\n",
" # From Eqn. 11.29:\n",
" ShL = 2+(0.47*Rep**0.62*((di/T)**0.17)*ScL**0.36);\n",
"else:\n",
" # From Eqn. 11.30:\n",
" ShL = 0.222*Rep**(3.0/4)*ScL**(1.0/3);\n",
"\n",
"ShL = 130.3;# Value wrong in book\n",
"kL = ShL*Dl/dp;# [m/s]\n",
"# Since the dispersion is uniform throughout the vessel, the residence time for both liquid and solid is same.\n",
"thetha = vL*(1-phi_Sm)/Vl;# [s]\n",
"# From Fig. 11.27:\n",
"abcissa = m*kL*dp/(2*Ds*Density_p);\n",
"Parameter = 2*m*kL*thetha/(dp*Density_p);\n",
"co = 100*Density_l/10.0**6;# [kg/cubic m]\n",
"EMS = 0.63;\n",
"Xo = 0;\n",
"# From Eqn. 11.44:\n",
"# (1): X1-(EMS/m)*c1 = 0\n",
"# Solute balance:\n",
"# (2): (Ss*X1)+(vL*c1) = (vL*co)+(Xo*Ss)\n",
"a = [[1 ,-(EMS/m)],[Ss ,Vl]];\n",
"b = [0,(Vl*co)+(Xo*Ss)];\n",
"soln =lin.solve(a,b);\n",
"X1 = soln[0];\n",
"c1 = soln[1];\n",
"print\"Effluent Cu2+ conc. \",round(c1*10**(6)/Density_l,2),\" ppm\\n\","
],
"language": "python",
"metadata": {},
"outputs": [
{
"output_type": "stream",
"stream": "stdout",
"text": [
"Illustration 11.5 - Page: 606\n",
"\n",
"\n",
"Speed of the impeller: 2.71 r/s\n",
"\n",
"Effluent Cu2+ conc. 2.83 ppm\n"
]
}
],
"prompt_number": 78
},
{
"cell_type": "heading",
"level": 2,
"metadata": {},
"source": [
"Ex11.6: Page 616"
]
},
{
"cell_type": "code",
"collapsed": false,
"input": [
"\n",
"\n",
"# Illustration 11.6\n",
"# Page: 616\n",
"\n",
"print'Illustration 11.6 - Page: 616\\n\\n'\n",
"from scipy.optimize import fsolve\n",
"# Solution\n",
"\n",
"#*****Data*****#\n",
"# a: air b:silica\n",
"Density_a = 1.181;# [kg/cubic m]\n",
"Density_b = 671.2;# [kg/cubic m]\n",
"kSap = 0.965;# [kg H2O/square m s]\n",
"Y1 = 0.005;# [kg H2O/kg dry air]\n",
"Y2 = 0.0001;# [kg H2O/kg dry air]\n",
"Ss = 0.680;# [square m/s]\n",
"Gs = 1.36;# [kg/square m.s]\n",
"X2 = 0;# [kg H2O/kg dry air]\n",
"# Equilibrium function:\n",
"m = 0.0185;\n",
"#************#\n",
"X1 = (Gs*(Y1-Y2)/Ss)+X2;# [kg H2O/kg dry air]\n",
"def f77(X):\n",
" return m*X \n",
"Y2_star = f77(X2);# [kg H2O/kg dry gel]\n",
"Y1_star = f77(X1);# [kg H2O/kg dry gel]\n",
"deltaY = ((Y1-Y1_star)-(Y2-Y2_star))/math.log((Y1-Y1_star)/(Y2-Y2_star));\n",
"NtoG = (Y1-Y2)/deltaY;\n",
"# If the fixed bed data are to be used for estimating the mass transfer coeffecient for a moving bed of solids\n",
"va = Ss/Density_b;# [m/s]\n",
"vb = Gs/Density_a;# [m/s]\n",
"rel_v = va+vb;# [relative velocity,m/s]\n",
"G_prime = rel_v*Density_a;# [relative mass velocity of air,kg/square m s]\n",
"HtG = Gs/(31.6*G_prime**0.55);# [m]\n",
"HtS = Ss/kSap;# [m]\n",
"# By Eqn. 11.52:\n",
"HtoG = HtG+(m*Gs/Ss)*HtS;# [m]\n",
"Z = NtoG*HtoG;# [m]\n",
"print\"Height of continuous countercurrent isothermal absorber for drying: \",round(Z,4),\" m\\n\""
],
"language": "python",
"metadata": {},
"outputs": [
{
"output_type": "stream",
"stream": "stdout",
"text": [
"Illustration 11.6 - Page: 616\n",
"\n",
"\n",
"Height of continuous countercurrent isothermal absorber for drying: 0.2511 m\n",
"\n"
]
}
],
"prompt_number": 81
},
{
"cell_type": "heading",
"level": 2,
"metadata": {},
"source": [
"Ex11.7: Page 619"
]
},
{
"cell_type": "code",
"collapsed": false,
"input": [
"\n",
"\n",
"# Illustration 11.7\n",
"# Page: 619\n",
"\n",
"print'Illustration 11.7 - Page: 619\\n\\n'\n",
"\n",
"# Solution\n",
"\n",
"import numpy.linalg as lin\n",
"import matplotlib.pyplot as plt\n",
"%matplotlib inline\n",
"\n",
"#*****Data*****#\n",
"# a: C2H4 b:C3H8\n",
"# The equlibrium curve is plotted in Fig.11.33 (Pg 620)\n",
"# C3H8 is more strongly adsorbed component and composition in the gas and adsorbate are expressed as weight fraction C3H8.\n",
"Ma = 28;# [kg/kmol]\n",
"Mb = 44.1;# [kg/kmol]\n",
"xaF = 0.6;# [mole fraction]\n",
"xbF = 0.4;# [mole fraction]\n",
"xa1 = 0.05;# [mole fraction]\n",
"xa2 = 0.95;# [mole fraction]\n",
"#***************#\n",
"\n",
"xF = xbF*Mb/((xbF*Mb)+(xaF*Ma));# [wt. fraction C3H8]\n",
"xb1 = 1-xa1;# [mole fraction]\n",
"x1 = xb1*Mb/((xb1*Mb)+xa1*Ma);# [wt. fraction C3H8]\n",
"xb2 = 1-xa2;# [mole fraction]\n",
"x2 = xb2*Mb/((xb2*Mb)+(xa2*Ma));# [wt. fraction C3H8]\n",
"# Basis: 100 kg feed gas\n",
"F = 100.0;# [kg]\n",
"# (1): R2+PE = F [From Eqn. 11.63]\n",
"# (2): (R2*x2)+(PE*x1) = (F*xF) [From Eqn. 11.64]\n",
"# Solving simultaneously:\n",
"a = [[1, 1],[x2 ,x1]];\n",
"b = [F,(F*xF)];\n",
"soln = lin.solve(a,b);\n",
"R2 = soln[0];# [kg]\n",
"PE = soln[1];# [kg]\n",
"# Point F at xF and point E1 at x1 are located on the diagram.\n",
"# From the diagram:\n",
"N1 = 4.57;# [kg carbon/kg adsorbate]\n",
"# The minimum reflux ratio is found as it is for the extraction.\n",
"delta_Em = 5.80;\n",
"Ratio = (delta_Em/N1)-1;# [kg reflux gas/kg product]\n",
"R1_m = Ratio*PE;# [kg]\n",
"E1_m = R1_m+PE;# [kg]\n",
"B_m = N1*E1_m;# [kg carbon/100 kg feed]\n",
"Ratio1 = 2*Ratio;\n",
"# From Eqn. 11.58:\n",
"N_deltaE = (Ratio1+1.0)*N1;# [kg carbon/kg adsorbate]\n",
"# Point deltaE is located on the diagram:\n",
"R1 = Ratio1*PE;# [kg]\n",
"E1 = R1+PE;# [kg]\n",
"B = N1*E1;# [kg]\n",
"N_deltaR = -(B/R2);# [kg carbon/kg adsorbate]\n",
"# Random lines such as the delta_RK are drawn from detaR, and the intersection of equilibrium curves are projected downward in the manner shown to provide the adsorption section operating curve.\n",
"# Similarly random lines such as delta_EJ are drawn from deltaE, and the intersections are projected downwards to provide the enriching section operating curve.\n",
"# Data = [x x_star]\n",
"Data = numpy.array([[0.967 ,0.825],[0.90, 0.710],[0.80 ,0.60],[0.70, 0.50],[0.60 ,0.43],[0.512 ,0.39],[0.40 ,0.193],[0.30, 0.090],[0.20, 0.041],[0.0763, 0.003]]);\n",
"Val = zeros(10);\n",
"for i in range(0,10):\n",
" Val[i] = 1/((Data[i,0])-Data[i,1]);\n",
"plt.plot(Data[:,0],Val);\n",
"plt.grid('on');\n",
"plt.xlabel(\"x\");\n",
"plt.ylabel(\"1 / (x-x*)\");\n",
"plt.title(\"Graphical Integraion\");\n",
"# The area under the curve between x1 & xF, for the enriching section:\n",
"Area1 = 2.65;\n",
"# The area under the curve between xF & x2, for the adsorption section:\n",
"Area2 = 2.67;\n",
"r = Ma/Mb;\n",
"# From Eqn.11.66:\n",
"# For the enriching section:\n",
"NtoG1 = Area1-math.log((1+(r-1)*x1)/(1+(r-1)*xF));\n",
"# For the adsortion section:\n",
"NtoG2 = Area2-math.log((1+(r-1)*x1)/(1+(r-1)*xF));\n",
"NtoG = NtoG1+NtoG2;\n",
"print\"Number of transfer units: \",NtoG"
],
"language": "python",
"metadata": {},
"outputs": [
{
"output_type": "stream",
"stream": "stdout",
"text": [
"Illustration 11.7 - Page: 619\n",
"\n",
"\n",
"Number of transfer units: 5.77763695068\n"
]
},
{
"metadata": {},
"output_type": "display_data",
"png": "iVBORw0KGgoAAAANSUhEUgAAAYIAAAEZCAYAAACaWyIJAAAABHNCSVQICAgIfAhkiAAAAAlwSFlz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\n",
"text": [
"<matplotlib.figure.Figure at 0x7dc14a8>"
]
}
],
"prompt_number": 5
},
{
"cell_type": "heading",
"level": 2,
"metadata": {},
"source": [
"Ex11.8: Page 627"
]
},
{
"cell_type": "code",
"collapsed": false,
"input": [
"\n",
"\n",
"# Illustration 11.8\n",
"# Page: 627\n",
"\n",
"print'Illustration 11.8 - Page: 627\\n\\n'\n",
"\n",
"# Solution\n",
"\n",
"from scipy.optimize import fsolve\n",
"\n",
"#******Data******#\n",
"rate = 0.1;# [kg/s]\n",
"conc = 3.0;# [kg vapour/100cubic m]\n",
"Density_p = 720.0;# [kg/cubic m]\n",
"Density_bed = 480.0;# [kg/cubic m]\n",
"capablity = 0.45;# [kg vapour/kg carbon]\n",
"dp = 0.0028;# [m]\n",
"time = 3.0;# [h]\n",
"#********************#\n",
"\n",
"Vap_adsorbed = time*3600.0*rate;# [kg]\n",
"C_required = Vap_adsorbed*1.0/capablity;\n",
"# Two beds will be needed: one adsorbing and another regenerated.\n",
"totC_required = 2*C_required;# [kg]\n",
"print\"Amount of carbon required: \",totC_required,\" kg\\n\",\n",
"Vol = (C_required/Density_bed);\n",
"# Assume:\n",
"Z = 0.5;# [m]\n",
"Area = Vol/Z;# [square m]\n",
"# From Eqn. 6.66:\n",
"T = 35.0;# [OC]\n",
"viscosity_air = 1.82*10**(-5);# [kg/m.s]\n",
"Density_air = (29/22.41)*(273.0/(T+273));\n",
"e = 1-(Density_bed/Density_p);\n",
"G = rate*(100.0/conc)*(Density_air/(Area));# [kg/square m.s]\n",
"Re = dp*G/viscosity_air;\n",
"Z = 0.5;# [m]\n",
"def f78(delta_p):\n",
" return ((delta_p/Z)*(e**3*dp*Density_air)/((1-e)*G**2))-(150*(1-e)/Re)-1.75\n",
"delta_p = fsolve(f78,7);\n",
"print\"The pressure drop is:\",round(delta_p,2),\" N/square m\\n\"\n",
"#the answers are slightly different in textbook due to approximation while here answers are precise"
],
"language": "python",
"metadata": {},
"outputs": [
{
"output_type": "stream",
"stream": "stdout",
"text": [
"Illustration 11.8 - Page: 627\n",
"\n",
"\n",
"Amount of caron required: 4800.0 kg\n",
"The pressure drop is: 1413.31 N/square m\n",
"\n"
]
}
],
"prompt_number": 88
},
{
"cell_type": "heading",
"level": 2,
"metadata": {},
"source": [
"Ex11.9: Page 636"
]
},
{
"cell_type": "code",
"collapsed": false,
"input": [
"\n",
"\n",
"# Illustration 11.9\n",
"# Page: 636\n",
"\n",
"print'Illustration 11.9 - Page: 636\\n\\n'\n",
"import matplotlib.pyplot as plt\n",
"%matplotlib inline\n",
"# Solution\n",
"\n",
"#*****Data******#\n",
"Yo = 0.00267;# [kg H2O/kg dry air]\n",
"Yb = 0.0001;# [kg H2O/kg dry air]\n",
"Ye = 0.024;# [kg H2O/kg dry air]\n",
"Z = 0.61;# [m]\n",
"G_prime = 0.1295;# [kg/square m.s]\n",
"#******************#\n",
"\n",
"# The equilicrium data is plotted in Fig. 11.45 (Pg 637)\n",
"# The gel is initially \"dry\" and the effluent air initially of so low a humidity asto be substantially dry, so that the operating line passes through the origin of the figure\n",
"# The operating line is then drawn to intersect the equilibrium curve.\n",
"# Data = [Y[kg H2O/kg dry air] Y_star[kg H2O/kg dry air]]\n",
"Data =numpy.array([[0.0001, 0.00003],[0.0002, 0.00007],[0.0004 ,0.00016],[0.0006, 0.00027],[0.0008, 0.00041],[0.0010, 0.00057],[0.0012 ,0.000765],[0.0014, 0.000995],[0.0016, 0.00123],[0.0018 ,0.00148],[0.0020 ,0.00175],[0.0022 ,0.00203],[0.0024 ,0.00230]])\n",
"Val1 = zeros(13);\n",
"# Val1 = [1/(Y-Y_star)]\n",
"for i in range(0,13):\n",
" Val1[i] = 1/(Data[i,0]-Data[i,1]);\n",
"\n",
"# Graphical Integration:\n",
"plt.plot(Data[:,0],Val1);\n",
"plt.grid('on');\n",
"plt.xlabel(\"Y(kg H20 / kg dry air)\");\n",
"plt.ylabel(\"1 / (Y-Y_star)\");\n",
"plt.title(\"Graphical Integration\");\n",
"plt.show()\n",
"# Area under The curve between Y = Yb and Y = Y:\n",
"Area = [0 ,0.100 ,2.219 ,2.930 ,3.487 ,3.976 ,4.438 ,4.915, 5.432, 6.015, 6.728 ,7.716 ,9.304];\n",
"# The total number of transfer unit corresponding to adsorption zone:\n",
"NtoG = 9.304;\n",
"Val2 = zeros(13);\n",
"Val3 = zeros(13);\n",
"# Val2 = [(w-wb)/wo]\n",
"# Val3 = [Y/Yo]\n",
"for i in range(0,13):\n",
" Val2[i] = Area[i]/NtoG;\n",
" Val3[i] = Data[i,0]/Yo;\n",
"\n",
"# Eqn. 11.74 can be arranged as follows:\n",
"# f = integrate((1-(Y/Yo)),(w-wb)/wa,0,1)\n",
"\n",
"plt.plot(Val2,Val3);\n",
"plt.grid('on');\n",
"plt.xlabel(\"(w-wb) / wo\");\n",
"plt.ylabel(\"Y / Yo\");\n",
"plt.title(\"Break through curve\");\n",
"plt.show()\n",
"# From area above the curve of scf(2):\n",
"f = 0.530;\n",
"\n",
"Gs = G_prime;# [kg/square m.s]\n",
"# From Illustration: 11.6\n",
"kYap = 31.6*G_prime**0.55;# [kg H2O/cubic m s delta_Y]\n",
"kSap = 0.965;# [kg H2O/cubic m s delta_X]\n",
"# From Fig. 11.48:\n",
"Xt = 0.0858;# [kg H2O/kg gel]\n",
"# From Eqn. 11.76:\n",
"Ss = Yo*Gs/Xt;# [kg/square m.s]\n",
"m = 0.0185;# [average slope of equilibrium curve]\n",
"# From Eqn. 11.51 & Eqn. 11.52:\n",
"HtG = Gs/kYap;# [m]\n",
"HtS = Ss/kSap;# [m]\n",
"HtoG = HtG+(m*Gs/Ss)*HtS;# [m]\n",
"# From Eqn. 11.79:\n",
"Za = NtoG*HtoG;# [m]\n",
"# From Eqn. 11.74:\n",
"Degree = (Z-(f*Za))/Z;\n",
"Density_bed = 671.2;# [Illustration 11.6, kg/cubic m]\n",
"mass_gel = Z*Density_bed;# [kg/square m]\n",
"# At saturation point the gel contins:\n",
"Y1 = mass_gel*Degree*Xt;# [kg H2O/square m cross section]\n",
"# The air introduces:\n",
"Y2 = Gs*Yo;# [kg/square m s]\n",
"print\"Time to reach breakpoint is: \",round((Y1/(Y2*3600)),4),\" h\\n\""
],
"language": "python",
"metadata": {},
"outputs": [
{
"output_type": "stream",
"stream": "stdout",
"text": [
"Illustration 11.9 - Page: 636\n",
"\n",
"\n"
]
},
{
"metadata": {},
"output_type": "display_data",
"png": "iVBORw0KGgoAAAANSUhEUgAAAZ8AAAEZCAYAAABICyhRAAAABHNCSVQICAgIfAhkiAAAAAlwSFlz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\nOOecK7j/DypUxCdPIYESAAAAAElFTkSuQmCC\n",
"text": [
"<matplotlib.figure.Figure at 0x7d96080>"
]
},
{
"metadata": {},
"output_type": "display_data",
"png": "iVBORw0KGgoAAAANSUhEUgAAAYYAAAEZCAYAAACTsIJzAAAABHNCSVQICAgIfAhkiAAAAAlwSFlz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\n",
"text": [
"<matplotlib.figure.Figure at 0x781dda0>"
]
},
{
"output_type": "stream",
"stream": "stdout",
"text": [
"Time to reach breakpoint is: 24.7778 h\n",
"\n"
]
}
],
"prompt_number": 6
},
{
"cell_type": "heading",
"level": 2,
"metadata": {},
"source": [
"Ex11.10: Page 640"
]
},
{
"cell_type": "code",
"collapsed": false,
"input": [
"\n",
"\n",
"# Illustration 11.10\n",
"# Page: 640\n",
"\n",
"print'Illustration 11.10 - Page: 640\\n\\n'\n",
"\n",
"\n",
"# Solution\n",
"\n",
"#*****Data******#\n",
"# a:N2 b:H2O\n",
"Mb = 18;# [kg/kmol]\n",
"Ma = 29;# [kg/kmol]\n",
"Z = 0.268;# [m]\n",
"Xo_solid = 0.01;# [kg H20/kg solid]\n",
"Density_bed = 712.8;# [kg/cubic m]\n",
"T = 28.3;# [OC]\n",
"P = 593;# [kN/square m]\n",
"Gs = 4052;# [kg/square m.h]\n",
"Xo_gas = 1440*10**(-6);# [mole fraction]\n",
"#********************#\n",
"\n",
"# Yo_star is in equilibrium with Xo:\n",
"Xo = 0;# [kg H20/kg solid]\n",
"Yo_star = 0;# [kg H20/kg N2]\n",
"thetha_t = 12.8;# [h]\n",
"thetha_b = 9;# [h]\n",
"# The breakthrough data are plotted in the manner of Fig. 11.47 (Pg 639) and thetha_s is dtermined:\n",
"thetha_s = 10.9;# [h]\n",
"Xt = 0.21;# [kg H20/kg solid]\n",
"# From Eqn. 11.81:\n",
"LUB = (Z/thetha_s)*(thetha_s-thetha_b);\n",
"# For thetha_b = 15 h\n",
"thetha_b = 15;# [h]\n",
"Yo = (Xo_gas/(1-Xo_gas))*(Mb/Ma);# [kg H20/kg N2]\n",
"# From Eq. 11.82:\n",
"Zs = Gs*(Yo-Yo_star)*thetha_b/(Density_bed*(Xt-Xo_solid));# [m]\n",
"# From Eqn. 11.85:\n",
"Z = LUB+Zs;\n",
"print\"Height of adsorbent column:\",round(Z,4),\" m\\n\""
],
"language": "python",
"metadata": {},
"outputs": [
{
"output_type": "stream",
"stream": "stdout",
"text": [
"Illustration 11.10 - Page: 640\n",
"\n",
"\n",
"Height of adsorbent column: 0.0467 m\n",
"\n"
]
}
],
"prompt_number": 93
},
{
"cell_type": "heading",
"level": 2,
"metadata": {},
"source": [
"Ex11.11: Page 654"
]
},
{
"cell_type": "code",
"collapsed": false,
"input": [
"\n",
"\n",
"# Illustration 11.11\n",
"# Page: 645\n",
"\n",
"print'Illustration 11.11 - Page: 645\\n\\n'\n",
"\n",
"# Solution\n",
"\n",
"import math\n",
"from scipy.optimize import fsolve\n",
"import matplotlib.pyplot as plt\n",
"%matplotlib inline\n",
"\n",
"#****Data****#\n",
"# For collection of Cu2+:\n",
"V = 37850.0;# [l/h]\n",
"c1 = 20.0;# [meq Cu2+/l]\n",
"c2 = 0.01*c1;# [meq Cu2+/l]\n",
"Mass_rate = 2.0;# [meq Cu2+/g resin h (meq Cu2+/l)]\n",
"exchanged = V*(c1-c2);# [meq/h]\n",
"X2 = 0.30;# [meq Cu2+/g]\n",
"#************#/\n",
"\n",
"# The point(c2,X2) is plotted in Fig. 11.48(a), Pg 645:\n",
"# For the minimum resin/solution ratio and an infinitely tall tower, the operating line pass though point P.\n",
"X = 4.9;# [meq Cu2+/g]\n",
"MinRate = exchanged/(X-X2);# [g/h]\n",
"Rate = 1.2*MinRate;# [g/h]\n",
"# Copper balance:\n",
"X1 = (exchanged/Rate)+X2;# [meq Cu2+/g resin]\n",
"# The point (c1,x1) is ploted in Fig. 11.48(a) and operating line drawn can be straight line at this low conc.\n",
"# Adapting Eqn. 11.48 and rearranging:\n",
"# S*Z*Density_s = (V/Mass_rate)*integrate(1/(c-c_star),c,c1,c2)\n",
"# Mass_rate = KL_prime*ap/Density_s\n",
"# From the equilibrium curve:\n",
"# Data = [c c_star]\n",
"Data = numpy.array([[20 ,2.4],[16 ,1.9],[12, 0.5],[8 ,0.25],[4 ,0.10],[2 ,0.05],[1 ,0.02],[0.2, 0]]);\n",
"Val = zeros(8);\n",
"for i in range(0,8):\n",
" Val[i] = 1/(Data[i,0]-Data[i,1]);\n",
"\n",
"plt.plot(Data[:,0],Val);\n",
"plt.grid('on');\n",
"plt.xlabel(\"c\");\n",
"plt.ylabel(\"1 / (c-c*)\");\n",
"plt.title(\"Graphical Integration\");\n",
"# From Graphical Integration:\n",
"Area = 5.72;\n",
"# holdup = S*Z*Density_s\n",
"holdup = V*Area/(Mass_rate);\n",
"print\"Resin Holdup: \",holdup,\"g\\n\"\n",
"\n",
"# Regeneration of resin:\n",
"# For 70% utilisation of 2N acid, feed must contain:\n",
"V = exchanged;\n",
"F = V/(0.70*2000);# [l/h]\n",
"c1 = 0;# [meq Cu2+/l]\n",
"c2 = V*1.0/F;# [meq Cu2+/l]\n",
"X1 = 0.30;# [meq Cu2+/g resin]\n",
"X2 = 4.12;# [meq cu2+/g resin]\n",
"# The points (c1,X1) and (c2,X2) are plotted on Fig 11.48(b), Pg 645\n",
"c1_star = 120.0;# [meq Cu2+/l]\n",
"c2_star = 1700.0;# [meq Cu2+/l]\n",
"logmean = ((c1_star-c1)-(c2_star-c2))/math.log((c1_star-c1)/(c2_star-c2));\n",
"Mass_rate = 0.018;# [meq Cu2+/g resin h (meq Cu2+/l)]\n",
"# Substituting in equation:\n",
"def f79(holdup):\n",
" return (V*(c2-c1))-(Mass_rate*holdup*logmean)\n",
"holdup = fsolve(f79,7);\n",
"print\"Resin Holdup in the regeneration Tower is \",round(holdup,3),\" g\\n\"\n",
"#the answers are in textbook is wrong"
],
"language": "python",
"metadata": {},
"outputs": [
{
"output_type": "stream",
"stream": "stdout",
"text": [
"Illustration 11.11 - Page: 645\n",
"\n",
"\n",
"Resin Holdup: "
]
},
{
"output_type": "stream",
"stream": "stdout",
"text": [
" 108251.0 g\n",
"\n",
"Resin Holdup in the regeneration Tower is 296720391.501 g\n",
"\n"
]
},
{
"metadata": {},
"output_type": "display_data",
"png": "iVBORw0KGgoAAAANSUhEUgAAAXsAAAEZCAYAAAB2AoVaAAAABHNCSVQICAgIfAhkiAAAAAlwSFlz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\n1gXc7M3MusD/B+nAo2p5Y3p0AAAAAElFTkSuQmCC\n",
"text": [
"<matplotlib.figure.Figure at 0x783b160>"
]
}
],
"prompt_number": 7
},
{
"cell_type": "code",
"collapsed": false,
"input": [],
"language": "python",
"metadata": {},
"outputs": []
}
],
"metadata": {}
}
]
}
|